LSAT and Law School Admissions Forum

Get expert LSAT preparation and law school admissions advice from PowerScore Test Preparation.

 Administrator
PowerScore Staff
  • PowerScore Staff
  • Posts: 8917
  • Joined: Feb 02, 2011
|
#27356
Complete Question Explanation

Point of Agreement—CE. The correct answer choice is (E)

Claude’s argument is complex, and contains two conclusions. The statement in the last two lines of the stimulus is the main conclusion, and the statement in the second half of the first sentence is a sub-conclusion. The two premises are introduced by the premise indicators “because” and “since.” Claude argues that the high number of middle-aged people in the workforce will result in fewer opportunities for promotion, which will then cause a decrease of incentive, leading to a decline of economic productivity and quality of life.

Thelma feels differently about the consequences of the “glut” of middle-aged workers. Her argument is that this will lead to many people starting their own companies, which will in turn increase productivity and the quality of life.

Answer choice (A): This is a Reverse answer. If the answer said, “Society’s economic productivity affects the quality of life in a society,” then both speakers would agree with this statement.

Answer choice (B): Claude does not address the “failure of many companies,” and so his view is unknown on this topic. Hence, he does not necessarily agree with this statement, and this answer choice is incorrect.

Answer choice (C): Thelma does not address the issue of “promotion,” and thus her view is unknown on this topic. Hence, she does not necessarily agree with this statement, and this answer choice is incorrect.

Answer choice (D): Neither speaker addresses what will happen to the size of the workforce in the coming years, and so this answer choice can be eliminated.

Answer choice (E): This is the correct answer choice. The argument of each speaker establishes that both believe that the number of middle-aged people in the workforce leads to certain situations that result in economic productivity changes. The fact that they believe the size of the workforce will have opposite economic productivity effects is irrelevant because the answer choice simply states that “economic productivity will be affected.”
 mkuo
  • Posts: 24
  • Joined: Nov 06, 2012
|
#6506
In this question looking for point of agreement of two people, the answer is (E).

But I don't understand, as I feel (C) is more appropriate. Doesn't Thelma's "glut of middle-age workers will lead..." mean the middle-aged workers who aren't working hard due to the lack of incentive (company promotion)? Hence this lack will motivate them to form their own companies.

What I'm trying to point out is doesn't she have to agree with that point in order to claim middle-age workers' motivation for forming own companies? My understanding is that she agrees with the first part of Claude's claim, but proposes an alternative consequence that has the opposite effect.

Explanation I got from Kaplan is "while (C) is integral to Claude's argument, Thelma has no opinion on it". Well yeah if she explicitly agrees with it then this wouldn't be much of a problem.

Help please?
User avatar
 Dave Killoran
PowerScore Staff
  • PowerScore Staff
  • Posts: 5852
  • Joined: Mar 25, 2011
|
#6523
Hi Mike,

Sure, we're glad to help :-D

I think you are reading too much into Thelma's statement to draw the inference that she agrees with the claim stated in answer choice (C). I believe that the way LSAC sees the "glut of middle-aged workers" comment she makes is as just a reference to the fact that there are a lot of middle-aged workers--too many, in fact. That is, it's just a numbers issue. The fact that they leave to form their own companies may mean that they don't see opportunities, hence they depart. To take that "glut" comment and then irretrievably connect it to the idea that how hard they work is a function of promotion opportunities goes too far. For example, it may be that in Thelma's view all these middle-aged workers work hard no matter what the promotion possibilities--some just strike out on their own to make more money, or have more power, etc. So, while I get the connection that you are trying to make, I think you can't guarantee that Thelma agrees with answer choice (C). Essentially, while it's possible she agrees with (C), it's not certain, and that kills the answer.

Please let me know if that helps. Thanks!
 mkuo
  • Posts: 24
  • Joined: Nov 06, 2012
|
#6525
Hi Dave,

Thanks for your explanation!

I think i can understand now how (C) might not be guaranteed in Thelma's statement.

Regarding (E), when I was working on this question I crossed it as a contender because the wording of it...

"Economic productivity will be affected by the number of middle-aged people in the workforce."

... felt as if it meant "economic productivity" will be affected by the changing number of middle-aged workers. And of course there was nothing about the effects on "economic productivity" regarding a relatively lower number (or even a different number) of middle-aged workers.

I think I might've prephrased it as
"Economic productivity will be affected by the relatively high number of ..."
or
"The relatively high number... will affect economic productivity..." etc etc

Am I reading too much into that answer as well? Or do you feel the answer might've been a bit flawed... or purposely worded that way because it is LSAT after...?

I hope I am not taking up too much of your time with these trivial questions.

Thank you again for clarifying my concerns! Greatly appreciated!

Regards,

Mike
User avatar
 Dave Killoran
PowerScore Staff
  • PowerScore Staff
  • Posts: 5852
  • Joined: Mar 25, 2011
|
#6537
Hi Mike,

No, no problem at all!

With (E), I do think you are reading too much into it. The way I read (E) is that even if the number of middle aged workers stays the same, just because there are so many of them (which they both agree on) there is going to be economic fallout. Claude thinks things get worse, and Thelma thinks things get better regardless.

I don't believe they were trying to be cute or clever with the wording here; as stated, I don't see any holes in (E) or any need to make the wording more specific. The question is, "does the number of middle-aged workers affect economic productivity?," and I think both speakers can be seen strongly supporting that idea. I think you agree with that, but something about the phrasing of "the number" in (E) just didn't sit well with you.

Please let me know if that helps. Thanks!
 mkuo
  • Posts: 24
  • Joined: Nov 06, 2012
|
#6541
Hi Dave,

It absolutely helped!

Thank you very much!
 lsatprep1215
  • Posts: 33
  • Joined: Dec 16, 2019
|
#74899
Hi, I know why E is correct but I want to ask what type of question is this? This is like the opposite of a disagreeing question?
 Paul Marsh
PowerScore Staff
  • PowerScore Staff
  • Posts: 290
  • Joined: Oct 15, 2019
|
#75289
Hi lsatprep1215! Nice job figuring this question out. As for your question, we call these ones "Point of Agreement" questions. Categorically, we lump them together with "Point at Issue" questions, because they're approached similarly. See: this PowerScore blog post where Point of Agreement and Point at Issue questions are grouped together: https://blog.powerscore.com/lsat/lsat-l ... ppearance/

The difference of course is that instead of finding the point about which the two people disagree, we're finding the point about which they agree. These questions are not as common as Point at Issue questions but they do pop up every now and again.

Hope that helps!

Get the most out of your LSAT Prep Plus subscription.

Analyze and track your performance with our Testing and Analytics Package.